What is the area of this figure

What Is The Area Of This Figure

Answers

Answer 1

Answer:

178m squared

Step-by-step explanation:

make 3 rectangular shapes

shape 1 ) 18×5=90

shape 2) 13×6= 78

shape 3) 5×2=10

Add up all the answers and that's the ans

90+78+10=178

Answer 2
The area is 178m squared

Related Questions

Find f(2) if f(x) = (x + 1)^2

Answers

Answer:

9

Step-by-step explanation:

Plug in 2 for x

You should get f(2)=(2+1)^2

Add 2+1 inside the parenthesis to get 3

Finally do (3)^2 to get 9

Write the equation in slope-intercept form. y = 6(x + 2) + 5x

Answers

Answer:

y=11x+12

Step-by-step explanation:

y = 6(x + 2) + 5x

y=6x+12+5x

y=11x+12

in slope interception form=  y=mx+c

                                                y=11x+12

Y=11x + 12 my brother

Answer fast please! I need an answer before 9:00!

Answers

Answer:

1536 cm^2 be the correct answ

Answer: Option 2

1536cm²

Lateral Area=32×(24/2)×4

=1536 (cm²)

Answered by Gauthmath must click thanks and mark brainliest

help please I'm doing some math homework ​

Answers

Answer:

Slope is 5/2

Step-by-step explanation:

[tex]{ \boxed{ \bf{slope = \frac{y _{2} - y _{1} }{x _{2} - x _{1}} }}}[/tex]

Substitute the terms:

[tex]{ \tt{slope = \frac{7 - ( - 3)}{4 - 0} }} \\ = \frac{5}{2} [/tex]

If [ -2 5 7 6] and |B| = -|A|, which matrix is matrix B?

Answers

|A| and |B| denote the determinants of the matrices A and B.

We have

[tex]|A| = \begin{vmatrix}-2&5\\7&6\end{vmatrix} = (-2)\times6-7\times5 = -12 - 35 = -47[/tex]

Then B is a matrix such that |B| = -|A| = +47.

The only choice satisfying this is C, since

[tex]\begin{vmatrix}12&1\\13&5\end{vmatrix} = 12\times5-13\times1 = 60 - 13 = 47[/tex]

To win the annual read-a-thon at his school, Scott has to read more books than any of his classmates over winter break. He starts by reading a lot of sports books. Then, he reads 10 mystery books. In all, Scott reads 23 books for the read-a-thon.

Answers

Answer:

13 sports books and 10 unknown books is 23 books

Step-by-step explanation:

we already know scott read 23 books total, we also know that he read 10 mystery books.

subtracting 10 unknown books from 23 total books, we get 13 sports books as your answer

I suck at math ik but i need anyones help please

Answers

Answer:

f(2)= 1

f-¹(1)= 2

f-¹(f(2))= 2

I hope I helped you^_^

How do I solve part c

Answers

Answer:

A possible answer could be 2√2. It depends where points B and C would be. I guess this information was given in the parts A and B.

Step-by-step explanation:

See picture attached

write an equation for a line that is perpendicular to 2x - 4y = 10

Answers

Answer:

5

The correct answer is Y= - __

4

Please help explanation if possible

Answers

Answer:

Step-by-step explanation:

Subtract 3x from both sides

3x - 3x - 2y = -3x + y

Combine

-2y = - 3x + 7  

Divide by - 2

-2y/-2 = -3x/-2 + 7/-2

y = 3x/2 - 7/2

A small bird can flap its wings 120 times in 30 s. What is the rate of change of wing flaps?

Answers

Answer:

4 Flaps per second

Step-by-step explanation:

M = 120/30

Answer:

4 times per second

Step-by-step explanation:

120 ÷ 30 = 4

30 ÷ 30 = 1

4 flaps per 1 second

so basically 4

Solve for x. Leave your answer in simplest radical form.

Answers

Answer:

2√5

Step-by-step explanation:

hypotenuse of the second triangle = √(9²-5²)

= 2√14

so,

x = √{(2√14)²-6²}

= 2√5

Answer:

2sqrt(5)

Step-by-step explanation:

The left triangle is a right triangle so we can use the Pythagorean theorem

a^2+b^2 = c^2 where a and b are the legs  and c is the hypotenuse

5^2 + b^2 = 9^2

b^2 = 9^2 - 5^2

b^2 = 81-25

b^2 =56

Taking the square root of each side

b = sqrt(56) = sqrt(4*14) = 2 sqrt(14)

The right side is also a right triangle so we can  use the Pythagorean theorem

a^2+b^2 = c^2 where a and b are the legs  and c is the hypotenuse

x^2 + 6^2 = (sqrt(56))^2

x^2 +36 = 56

x^2 = 56-36

x^2 = 20

Taking the square root

sqrt(x^2) = sqrt(20) = sqrt(4*5) = 2sqrt(5)

hi, please solve these three questions for me, i have to shoe solving steps.​

Answers

question 3

Step-by-step explanation:

i only able to show you the step of question 3..so sorry

What is the value of the expression 3y−z
y+5z
when y = 6 and z = 2?

Answers

[tex]\huge\text{Hey there!}[/tex]

[tex]\huge\text{Question \#1}[/tex]

[tex]\huge\textsf{3y - z}[/tex]

[tex]\huge\textsf{= 3(6) - 2}[/tex]

[tex]\huge\textsf{= 18 - 2}[/tex]

[tex]\huge\textsf{= 16}[/tex]

[tex]\boxed{\boxed{\huge\textsf{Answer: 16}}}\huge\checkmark[/tex]

[tex]\huge\text{Question \#2}[/tex]

[tex]\huge\textsf{y + 5z}[/tex]

[tex]\huge\textsf{= 6 + 5(2)}[/tex]

[tex]\huge\textsf{= 6 + 10}[/tex]

[tex]\huge\textsf{= 16}[/tex]

[tex]\boxed{\boxed{\huge\textsf{Answer: 16}}}\huge\checkmark[/tex]

[tex]\huge\textsf{\boxed{\star\underline{\underline{16 = 16}}\star}\star}[/tex]

[tex]\huge\text{Good luck on your assignment \& enjoy your day!}[/tex]

~[tex]\frak{Amphitrite1040:)}[/tex]

(PICTURE) Im really struggling with questions like these at the moment, if you could please help me out thank you

Answers

Answer:

Arthur should select the Mount Joy Pool when there are less than 10 people at the party

Step-by-step explanation:

The initial fee to rent the Woodbridge Pool = $50

The additional fee per person after renting = $5

The initial fee to rent at Mount Joy Pool = 0 (no initial fee)

The fee per person at Mount Joy Pool = $10

The equation of a straight line is y = m·x + c

Ley y  represent the total cost of renting the pool, x, represent the number of persons in the pool, m represent the fee per person and c represent the initial charges, we get;

The total cost of renting Woodbridge Pool, y = 5·x + 50

The total cost of renting the Joy Pool, y = 10·x

Equating both values of y gives;

5·x + 50 = 10·x

∴ 5·x = 50

x = 50/5 = 10

x = 10

From the above equations, the cost of renting the pool is lower for the Joy Pool when there are less than 10 persons at the pool

It will cost the same amount to rent the pool when the number of persons in the pool are 10 persons and the cost of renting the Mount Joy Pool  will be more than the cost of renting the Woodbridge Pool, when there are more than 10 persons

Therefore, Arthur should select the Mount Joy Pool when there are less than 10 people at the party.

PLzzZZHELPPPP


The angle between the top of a building and a point 80 feet away from the base (on level ground) is 70° . To the nearest foot, how tall is the building?

Answers

Answer:

220ft tall

Step-by-step explanation:

From the question we are given the following:

Adjacent side of the triangle = 80feet

Angle of elevation = 70°

Required

Height of the building (Opposite)

Using the SOH CAH TOA identity:

tan theta = opp/adj

tan 70 = H/80

H = 80tan70

H = 80(2.7474)

H = 219.79ft

Hence the building is 220ft tall

I NEED HELP WITH THIS QUESTION ASAP ITS REALLY REALLY URGENT!!!!


Two kids, Albert and Bhara, are 20.0 m apart. Albert sees a soccer ball 25.0 m away. If the angle between the line formed by Albert and Bhara and the line from Albert to the soccer ball is 25 degrees, how far is Bhara from the soccer ball? Correctly round your answer to the nearest tenth of a meter.

Answers

Step-by-step explanation:

I can give you a formula SOH-Sinø=Opposite\Hypotenuse

CAH-Cos=Adjacent\Hypotenuse

TOA-Tan=Opposite\Adjacent

A train travels a distance of 60 km at uniform speed. If the speed of the train was reduced by 10 kmh-1, the time taken to travel the 60km will increase by 1/2h. Find the speed of the train at the beginning.

Answers

Answer:

Initial speed is 32 m/s

At uniform speed, acceleration is 0, (a = 0).

When speed reduced, (v - u) = 2.78 ms-¹, t = 1800 sec, s = 60 ,000 metres.

From first equation of motion:

[tex]{ \boxed{ \bf{v = u + at}}} \\ { \tt{(v - u) = at}}[/tex]

substitute:

[tex]{ \tt{2.78 = (a \times 1800)}} \\ { \tt{acceleration = 0.0015 \: {ms}^{ - 2} }}[/tex]

from second equation of motion:

[tex]{ \boxed{ \bf{s = ut + \frac{1}{2} a {t}^{2} }}}[/tex]

substitute:

[tex]{ \tt{60000 = 1800u + ( \frac{1}{2} \times 0.0015 \times {1800}^{2}) }} \\ { \tt{1800u = 57570}} \\ { \tt{u = 32 \: m {s}^{ - 1} }}[/tex]

Math models helpppp plss if you know about math models answer this pls

Answers

Answer:

A mathematical model is a description of a system using mathematical concepts and language. The process of developing a mathematical model is termed mathematical modeling. Mathematical models are used in the natural sciences and engineering disciplines, as well as in non-physical systems such as the social sciences.

A bus has velocity 20m/s towards east and another bus has velocity 15m/s in West direction.If they start to move from a point simultaneously.What distances do they covered in 2 minutes.​

Answers

Answer:

4,200 m or 4.2 km.

First bus: 2400 m

2nd bus: 1800 m.

Step-by-step explanation:

Their combines speed is 20+15 = 35m/s.

Distance = speed * time

2 minutes = 120 seconds so

D = 35 * 120

= 4200 m.

Splitting the distances:

The first bus covers 20 * 120 = 2400m

The second bus covers 15*120 = 1800m.

Answer:

4200m

Step-by-step explanation:

If two objects A and B are moving in the opposite direction with speed xm/s and ym/s respectively,then;

Relative speed =(x+y)m/s

: Relative speed = 20+15=35m/s

time =2min=160sec

DISTANCE=SPEED ×TIME

D=35×120=4200

4200m

A 60 foot monument casts a 25-foot shadow How long is the shadow of a 4.5 tall person?
A) 1.875 feet (it’s this one)
B) 5.555 feet
C) 2456 feet
D) 13334 feet

Answers

Answer:

A

Step-by-step explanation:

Using similarities we have

(Actual size)/(Shadow length)=60/25=4.5/x

x=4.5*25/60=1.875

Answer:

A) 1.875 feet

Step-by-step explanation:

Which inequality is represented by the graph?

Answers

Answer:

It will be -5≤×≤4

Will Mark Brainlest Help pls​

Answers

Answer:

x=2,y=2

Step-by-step explanation:

3y-1 = 5

or, 3y = 5+1

or, y = 6/3

y = 2

2x+1 = 3y-1

or, 2x = 3×2-1-1

or, 2x = 4

or, x = 2

[tex]\\ \sf\longmapsto \left[{2x+1\atop 2x+x}\:\:\:{3y-1\atop 3}\right]=\left[{3y-1\atop 4}\:\:\;{5\atop 3}\right][/tex]

[tex]\\ \sf\longmapsto 3y-1=5[/tex]

[tex]\\ \sf\longmapsto 3y=5+1[/tex]

[tex]\\ \sf\longmapsto 3y=6[/tex]

[tex]\\ \sf\longmapsto y=2[/tex]

Put the value in 1st one

[tex]\\ \sf\longmapsto 2x+1=3y-1[/tex]

[tex]\\ \sf\longmapsto 2x+1=3(2)-1[/tex]

[tex]\\ \sf\longmapsto 2x+1=6-1[/tex]

[tex]\\ \sf\longmapsto 2x+1=5[/tex]

[tex]\\ \sf\longmapsto 2x=5-1[/tex]

[tex]\\ \sf\longmapsto 2x=4[/tex]

[tex]\\ \sf\longmapsto x=\dfrac{4}{2}[/tex]

[tex]\\ \sf\longmapsto x=2[/tex]

Jeanette ice cream shop sold 10 sundaes with nuts and 7 sundaes without nuts to the total number of sundaes

Answers

The answer to the question is 17
There are 10 sundaes with nuts, and 7 without nuts. Our task is to find the total number of sundaes. We can find this by adding 10 + 7. This gives us 17 sundaes.

From an 80ft building the angle of elevation of the top of a taller building is 59 degrees and the angle of depression of the base of this building is 65 degrees determine the height of the taller building

Answers

Answer:

First we find the distance between the two buildings =

Step-by-step explanation:

[tex] \frac{80}{ \tan(65) } = 37.3 ft[/tex]

then the upper part of the taller building =

[tex]37.3 \times \tan(59) = 62 \: ft[/tex]

Now the total height of the taller building

[tex]80 + 62 = 142 \: ft[/tex]

Five consecutive multiples of 7 have a sum of 350. What is the smallest of these numbers?
A. 70
B. 56
C. 77
D. 84

Answers

Answer:

B. 56

Step-by-step explanation:

x + (x + 7 ) + ( x + 14 ) + ( x + 21 ) + ( x + 28 ) = 350

( x + x + x + x + x ) + ( 7 + 14 + 21 + 28 ) = 350

5x + 70 = 350

     - 70    - 70

_____________

5x = 280

x = 56

Hope this helps!

The smallest number is 56, the correct option is B.

What is Sum?

The sum is the output of the mathematical operation, Addition.

Let the first number is x, they are multiples of 7,

As they are multiples of 7, the consecutive numbers will be added by 7 for next term.

The 5 consecutive numbers can be written as x, (x+7), (x+14), (x+21), (x+28).

The equation can be formed for the numbers that are given as,

An equation is a mathematical statement that relates an algebraic expression with other expression by an equal sign.

The sum of the multiples is 350

x + (x + 7 ) + ( x + 14 ) + ( x + 21 ) + ( x + 28 ) = 350

Grouping the variables and the constants separately

( x + x + x + x + x ) + ( 7 + 14 + 21 + 28 ) = 350

5x + 70 = 350

Adding (-70) to both sides of the equation

5x = 280

Dividing both sides by 5

x = 56

The value of the first number of the series is obtained.

The first value is the smallest number of the series.

To know more about Sum

https://brainly.com/question/13013054

#SPJ2

Will mark brainlest helppppppp​

Answers

Answer:

6

Step-by-step explanation:

again ?

7 = (3x - 4)/2

14 = 3x - 4

18 = 3x

x = 6

What is the number when 4is subtracted from the Sum of 4 and 7 multiplied by one-fourth of 20?



Answers

Answer:

one fourth of 20 = 5

(4+7)-4*5

11-4*5

11-20

-9

y is inversely proportional to V1+x.
When x = 8, y = 2.
Find y when x = 15.

Answers

Answer:

y = 1.5

Step-by-step explanation:

Given x is inversely proportional to [tex]\sqrt{1+x}[/tex] , then the equation relating them is

y = [tex]\frac{k}{\sqrt{1+x} }[/tex] ← k is the constant of proportion

To find k use the condition when x = 8, y = 2

2 = [tex]\frac{k}{\sqrt{1+8} }[/tex] = [tex]\frac{k}{\sqrt{9} }[/tex] = [tex]\frac{k}{3}[/tex] ( multiply both sides by 3 )

6 = k

y = [tex]\frac{6}{\sqrt{1+x} }[/tex] ← equation of proportion

When x = 15 , then

y = [tex]\frac{6}{\sqrt{1+15} }[/tex] = [tex]\frac{6}{\sqrt{16} }[/tex] = [tex]\frac{6}{4}[/tex] = [tex]\frac{3}{2}[/tex] = 1.5

help me with the picture please

Answers

Answer:  115

=====================================================

Explanation:

Refer to the diagram below. I've drawn diagonal that slopes upward. This diagonal cuts the quadrilateral into two triangles: One is equilateral and the other is isosceles.

The equilateral triangle marked in blue has all three angles 60 degrees each.

Note that the 60 and y angles combined to form 130, so,

60+y = 130

y = 130-60

y = 70

Then focus on the isosceles triangle (angles y, w and w). These three interior angles must add to 180

y+w+w = 180

70+2w = 180

2w = 180-70

2w = 110

w = 110/2

w = 55

This adds onto its adjacent neighbor of 60 to get w+60 = 55+60 = 115 degrees which is the value of x.

Other Questions
Which graph shows the solution to y< -x+3? A generator uses a coil that has 270 turns and a 0.48-T magnetic field. The frequency of this generator is 60.0 Hz, and its emf has an rms value of 120 V. Assuming that each turn of the coil is a square (an approximation), determine the length of the wire from which the coil is made. PLEASE HELP!! Important test!! Option 1 isn't coming up so if you could try to solve others that would be great! what is the answer? I need help!! please and thank you Stuart needs to run by the store on his way to school. He begins at home, which on a coordinate plane is represented by the point (0,4). He walks to the store, which is the equivalent of walking 2 units right and 4 units down. After grabbing something to eat for lunch, he walks 3 units to the left and 5 units up to school. What point represents the school solve the equation:|x+3| >2x-1 does anyone on here use FLVS?? If so can u help with comprehensive science 5.06 conclusion part? Strategic alliances are more likely to be long lasting-when Group of answer choices both partners are experienced with strategic alliances and routinely enter into collaborative agreements with firms in peripheral industries. the alliance involves partners based in countries with distinctly different cultures and consumer buying habits and preferences. each partner has considerable resource weaknesses in the marketplace. they involve collaboration with suppliers or distribution allies or when both parties conclude that continued collaboration is in their mutual interests. the alliance involves joining forces in R What is environmental resistance? How does it affect populations? Which of the following words is generally used to describe what managers do as opposed to what leaders do b) Organize c) Inspire O d) Innovate One of the following must be used as a starting point in order to perform an analysis that will determine what the connections between imbalances of trade in goods and services and the flows of international financial capital are. Which one is it? A. sketch patterns of trade deficits B. sketch patterns of trade surpluses C. define the level of trade D. define the balance of trade linear equation 7x+25=9 What did the slave ships transport in the last leg of the triangular trade path? A 26-year-old G2P1 woman at 41 weeks gestation is brought in by ambulance. The emergency medical technician reports that a pelvic examination performed 20 minutes ago when the patient had a severe urge to push revealed that she was fully dilated and the fetal station was 2. Fetal heart tones were confirmed to be in the 150s, with no audible decelerations. When the patient is placed on the fetal monitor, the heart rate is noted to be in the 60s. The maternal heart rate is recorded as 100. Without pushing, the fetal scalp is visible at the introitus. A repeat pelvic exam shows that the infant is in the occiput anterior position. What is the most appropriate next step in the management of this patient The method of tree-ring dating gave the following years A.D. for an archaeological excavation site. Assume that the population of x values has an approximately normal distribution. 1,2851,1871,2221,1941,2681,3161,2751,3171,275Required:a. Use a calculator with mean and standard deviation keys to find the sample mean year x and sample standard deviations.b. Find a 90% confidence interval for the mean of all tree-ring dates from this archaeological site. (Round your answers to the nearest whole number.) When an object falls freely, the formula for the distance it falls is d= 4.9t^2 where d isthe distance, in metres, and t is time, in seconds.a) After 5 seconds, how far does an object fall, in metres? b) How many seconds does it take to fall 490 m? Tyler, a citizen of Utah, files a lawsuit in federal court against MegaCorp. Tyler's claim is based upon a violation of federal law. The federal district court has original jurisdiction, which means that Group of answer choices federal cases usually originate in federal district courts. the court has unusual procedural rules. the court has a unique method of deciding whether to hear a case. the subject matter of the suit is interesting and new. 10,4,7The above set 4 numbers have a mean of 8what could the mystery number which of the following is not good conductor of heat? 1-brass2-stainless steel 3-bakelite 4-silver Find hypotenuse,perpendicular and base